Howdy, Stranger!

It looks like you're new here. If you want to get involved, click one of these buttons!

Weakening Q AC characteristics

Is it possible to have 2 answer choice that both weaken the argument but to different degree?

Also, for the weakening question in general, if I have to make an additional assumption to believe one of the AC could possibly weaken, then it would generally be wrong, right? Perhaps subtle ones don't necessarily do this..

Am I in the right mindset for this? I would really appreciate if someone could confirm my thoughts on this.

Comments

  • tjphilbricktjphilbrick Alum Member
    174 karma

    Yes, that is possible, and the one that weakens it to the greater degree would be correct since they always ask for the AC that "most weakens" the argument. Also keep an eye out for AC's that weaken a part of the argument that isn't central to the reasoning. For instance, an AC could weaken a part of the argument that is totally irrelevant to the reasoning structure and therefore isn't even applicable.

  • slee146slee146 Member
    42 karma

    Thank you!! :)

  • Cant Get RightCant Get Right Yearly + Live Member Sage 🍌 7Sage Tutor
    27872 karma

    I don't recall ever having seen a Weaken question with two answer choices that weaken. For every Weaken question I've come across, there has been one AC that weakens and four that do not weaken in the slightest. And if you need additional assumptions to make one work, then that probably should disqualify it. Just make sure you're not dismissing a common sense implication of the AC by mistaking it for an inappropriate assumption. Sometimes it can feel like a difficult distinction!

  • slee146slee146 Member
    42 karma

    YES!! That really is a tough distinction. Ever since I started studying, I did not feel so comfortable distinguishing what is a reasonable common sense assumption vs. unsupported stretch.
    More problems I see, I think I am getting vague sense of how to tell but it still gets me not only in LR but in RC as well especially inference or what author most likely to agree with type of Qs..
    Adjusting level of suspicion is not so natural for me at least.

    Thanks for the advice!

  • JustDoItJustDoIt Alum Member
    3112 karma

    Hi!

    I think what really helped me was going back over the Causation Hypothesis lessons. It really did show that answer choices are wildly irrelevant and like @"Cant Get Right" said, there is always only right answer choice.

    I think the reason that they say "most weakens" is a CYA move just in case someone complains and says another answer choice weakens too. LSAC can then come back and grant that true but still ultimately conclude that the right answer weakens "more."

  • Cant Get RightCant Get Right Yearly + Live Member Sage 🍌 7Sage Tutor
    27872 karma

    @JustDoIt said:
    I think the reason that they say "most weakens" is a CYA move just in case someone complains and says another answer choice weakens too. LSAC can then come back and grant that true but still ultimately conclude that the right answer weakens "more."

    Yes. That's exactly why they include the "most."

Sign In or Register to comment.